Prueba matemática de la Segunda Ley de la Termodinámica [duplicado]

¿Puedes formalizar la mecánica estadística, como algunas personas han hecho con la relatividad, y probar la segunda ley de la termodinámica a partir de axiomas más fundamentales?

No creo que exista una prueba verdadera; las controversias enumeradas en la página de wikipedia no se han resuelto por completo. En particular, el teorema de recurrencia de Poincaré dice que, si esperas lo suficiente, la segunda ley de la termodinámica eventualmente fallará . Sin embargo, como señala el artículo, esto sucede con tan poca frecuencia que no vale la pena preocuparse. Aún así, cosas como esa impiden una prueba totalmente rigurosa.

Respuestas (4)

Creo que fue Boltzmann quien primero hizo la conexión entre la entropía y los microestados. el capítulo 12 de "Termodinámica clásica y estadística" de Ashley H. Carter analiza los argumentos de Boltzmann. Para resumir de ese libro:

Entropía ( S ) corresponde a una configuración particular de un conjunto de partículas llamado macroestado. Un estado macro se puede lograr usando varios estados micro diferentes ( w ). Por lo tanto, S = F ( w ) . Los estados micro representan la probabilidad de estar en el estado macro (solo necesitan normalizarse). Si se combinan dos sistemas, la entropía total es simplemente S = S A + S B , o F ( w ) = F ( w A ) + F ( w B ) . La probabilidad de estar en micro estado w es solo w A w B , ya que las probabilidades independientes son multiplicativas. Por lo tanto, S ( w A w B ) = S ( w A ) + S ( w B ) . Carter luego dice que la única función que satisface esta propiedad es el logaritmo natural, por lo que S yo norte ( w ) . La constante de proporcionalidad es k B . Muchos más detalles en el texto y ejemplos de microestados (discusión también en el contexto cuántico, que conduce a la densidad de estados).

El libro tiene un buen tratamiento general de la mecánica estadística. Comienza con la termodinámica clásica y luego pasa a la mecánica estadística y hace la conexión con la mecánica cuántica.

Este es un problema muy difícil, trato de explicar por qué.

En mecánica estadística, tender a la distribución más probable es un evento de probabilidad, y para la entropía de Boltzmann, d S 0 es también un evento de probabilidad pero no un resultado inevitable. Así que no puedes probar d S 0 como resultado inevitable de la mecánica estadística.

Si queremos obtener la prueba matemática de la segunda ley de la termodinámica, debemos considerar primero la prueba matemática de la entropía, como una función de estado. definicion de clausius d S = d q / T no se puede demostrar en matemáticas, como un diferencial exacto, por lo que la definición d S = d q / T sólo puede depender de ciclos reversibles imaginarios. Por otro lado, en los enfoques de C.Carath´eodory o M.Planck, las expresiones de la entropía son las ecuaciones matemáticas pero no la definición de un concepto físico porque la ecuación contiene la diferencia de funciones (ver más abajo la ecuación de Euler) , la imagen física de la entropía y la segunda ley no son claras, por eso no podemos explicar el significado físico de la entropía de acuerdo con estos enfoques. En tal caso, demostrar la segunda ley en matemáticas será muy difícil, no es un problema aislado.

Los siguientes son algunos pasos del documento de enlace, el documento presenta un nuevo enfoque y la nueva declaración sobre la segunda ley puede considerarse como un axioma.

1) Según la ecuación fundamental de la termodinámica (ecuación de Euler).

d S = d tu T Y d X T j m j d norte j T + pag d V T .

2) Defina la función que

d q = d tu Y d X j m j d norte j .

Aquí d q puede demostrarse como un diferencial exacto en matemáticas, el significado físico de q es la energía térmica dentro del sistema. (pero no el calor en la transferencia q )

3) Tal que obtenemos

d S = d q T + pag d V T .

Aquí d S puede demostrarse como un diferencial exacto en matemáticas.

4) Considere una interacción entre los dos locales, luego podemos obtener el diferencial total de la producción de entropía.

d i S = ( 1 T ) d q + 1 T Y d X + j 1 T m j d norte j + ( pag T ) d V .

Esta es una ecuación termodinámica de no equilibrio.

5) Demostrar el diferencial total de la producción de entropía d i S 0 .

El nuevo enunciado de la segunda ley: " la irreversibilidad radica en un principio fundamental: los gradientes de las cuatro fuerzas termodinámicas tienden espontáneamente a cero ".

Los cuatro gradientes de fuerzas termodinámicas son

( 1 T ) , Y , m j , ( pag T ) .

Las condiciones de equilibrio termodinámico son estos cuatro gradientes iguales a cero.

Compare las diferentes declaraciones sobre la segunda ley y vea qué declaración se puede considerar como un axioma. 1)-4) se citan de http://en.wikipedia.org/wiki/Second_law_of_thermodynamics

1) Declaración de Clausius

  • El calor nunca puede pasar de un cuerpo más frío a uno más caliente sin algún otro cambio, relacionado con él, que ocurra al mismo tiempo. El calor nunca puede pasar de un cuerpo más frío a uno más caliente sin algún otro cambio, relacionado con él, que ocurra al mismo tiempo.

2) Declaración de Kelvin

  • Es imposible, por medio de la acción material inanimada, derivar un efecto mecánico de cualquier porción de materia enfriándola por debajo de la temperatura del más frío de los objetos circundantes.

3) la declaración de Planck

  • Todo proceso que ocurre en la naturaleza procede en el sentido de que aumenta la suma de las entropías de todos los cuerpos que toman parte en el proceso. En el límite, es decir, para procesos reversibles, la suma de las entropías permanece invariable.

4) Principio de Carathéodory

  • En todo entorno de cualquier estado S de un sistema adiabáticamente cerrado hay estados inaccesibles desde S.

5) La nueva declaración.

  • Los gradientes de las cuatro fuerzas termodinámicas tienden espontáneamente a cero.

Nota

  • La mecánica estadística se restringe al postulado de igual probabilidad a priori, pero este postulado no necesita ser considerado en termodinámica, por lo que los rangos válidos de las dos teorías son diferentes. El rango válido del teorema H es menor que la segunda ley de la termodinámica. Hay algunas simulaciones por computadora para el teorema H, los cambios en H no son monótonos.

Por favor mira

http://arxiv.org/pdf/1201.4284v5.pdf

Puede editar su respuesta anterior (ya aceptada). Está mal visto publicar múltiples respuestas a menos que sean realmente muy diferentes.
@Freeman "la imagen física de la entropía y la segunda ley no están claras, ..." Esto no es del todo cierto. La imagen física generalmente tiene que ver con estados de partículas (estadísticas, la ecuación de Boltzmann y variaciones de la misma) que se conecta fácilmente con la mecánica cuántica y puede usarse para derivar ecuaciones termodinámicas (por ejemplo, diferentes ecuaciones de estado). Su derivación anterior se deriva de la termodinámica macroscópica (clásica). Como tal, es difícil obtener una visión física a ese nivel.
@ user3814483, no lo creo, hay algunos problemas que difícilmente pueden explicarse solo con mecánica estadística, por ejemplo, con la ecuación de Euler, Y X y m norte no contribuyen a la entropía, por lo que los números de los grados de libertad/estados en Y X y ambos m norte no tiene sentido calcular la entropía, ¿puedes distinguir estas dos partes de la energía total del sistema? mi y la energía de las partículas ε para un sistema termodinámico general? Si no, la imagen física de la entropía y la segunda ley seguirán sin estar claras.
Esto parece ser un enchufe para el periódico vinculado, que tiene todas las características de ser un periódico chiflado: hace afirmaciones "gee whiz"; tiene referencias solo a documentos antiguos en lugar de trabajos actuales en el campo; no ha sido publicado en una revista revisada por pares; y está escrito en un inglés ininteligible.
@Ben Crowell, no me importan sus comentarios, pero "Prueba matemática de la segunda ley de la termodinámica" y "¿Cómo se prueba la segunda ley de la termodinámica a partir de la mecánica estadística?" son las dos preguntas diferentes, la primera pregunta incluye el enfoque termodinámico, pero la segunda pregunta no. Observé su respuesta para la segunda pregunta: si cree que, y estoy de acuerdo, "no puede" probar la segunda ley de la termodinámica a partir de la mecánica estadística, ¿por qué no intentarlo con el enfoque termodinámico?
@Ben Crowell, ACuriousMind, Bernhard, Qmechanic, "Prueba matemática de la segunda ley de la termodinámica" y "¿Cómo se prueba la segunda ley de la termodinámica a partir de la mecánica estadística?" son las dos preguntas diferentes.

Hay un nuevo enunciado sobre la segunda ley: "la irreversibilidad tiene su raíz en un principio fundamental: los gradientes de las cuatro fuerzas termodinámicas tienden espontáneamente a cero". Por favor mira

http://arxiv.org/abs/1201.4284

No estoy seguro de por qué se aceptó esta respuesta. No formaliza la mecánica estadística ni la discute realmente en profundidad en el contexto de la segunda ley.

Creo que la segunda ley de la termodinámica es una consecuencia directa de la simetría del espacio-tiempo. Imagina que tienes dos cubos idénticos de metal A y B con un lado común, el primero más cálido que el otro. Si esperas lo suficiente, A y B terminarán estando a la misma temperatura. El mismo fenómeno ocurriría si B estuviera más caliente que A. Ahora imagina que inviertes el tiempo, de modo que de dos cubos a la misma temperatura, terminas con uno más caliente que el otro. Pero, ¿por qué la naturaleza elegiría arbitrariamente entre A y B? Esto implicaría que las leyes de la naturaleza dependen de la dirección del espacio que consideres, lo que sería contradictorio con la relatividad.

Es fácil inventar ejemplos similares que violen la segunda ley pero no violen la simetría espacial. Por ejemplo, una esfera inicialmente a temperatura uniforme podría desarrollar un gradiente de temperatura radial.
Lo que quiero decir aquí es que si la segunda ley de la termodinámica fuera falsa, habría un caso de violación de esta ley que estaría en contradicción con la relatividad.
"las leyes de la naturaleza dependen de la dirección del espacio que se considere, lo que sería contradictorio con la relatividad". No tiene sentido. ¿Cuál es la dirección del espacio?